Download as pdf or txt
Download as pdf or txt
You are on page 1of 13

Mid-term Revision

MCQ:
1
1. A firm has a total cost function of C(Q) = 75 + 25𝑄 2 . The firm experiences
a. Economies of scale
b. Diseconomies of scale
c. Constant returns to scale
d. All the statements associated with this question are correct.
2. Suppose two types of consumers buy suits. Consumers of type A will pay $100 for a coat,
and $50 for pants. Consumers of type B will pay $75 for a coat, and $75 for pants. The firm selling
suits faces no competition and has a marginal cost of zero. If the firm can identify each consumer
type and can price discriminate, what is the optimal price for a pair of pants?
a. Charge both types $150
b. Charge both types $75
c. Charge type A consumers $50, and type B consumers $75
d. Charge type A consumers $50, and type B consumers $50

3. First-degree price discrimination


a. Occurs when a firm charges each consumer the maximum price he or she would be
willing to pay for each unit of the good purchased
b. Results in the firm extracting all surplus from consumers
c. Occurs when a firm charges each consumer the maximum price he or she would be
willing to pay for each unit of the good purchased and results in the firm extracting all
surplus from consumers
d. None of the statements associated with this question are correct

4. Suppose that the demand for a monopolist's product is estimated to be Qd = 100 - 2P and
its total costs are C(Q) = 10Q. Under first-degree price discrimination the optimal number of
total units exchanged, profit and consumer surplus are
a. Q = 40, profit = $800; CS = $400
b. Q = 80; profit = $1600; CS = $1600
c. Q = 80, profit = $1600; CS = $0
d. Q = 40, profit = $600; CS = $0

5. The price elasticity of demand for senior citizens purchasing coffee from Brazilian Coffee is
-5 while non senior citizens have a price elasticity of demand equal to -1.25. If the marginal
cost of producing coffee is $0.02, the optimal price for a cup of coffee for senior citizens and
the optimal price for a cup of coffee for non-senior citizens are, respectively,
a. $0.016 and $0.20
b. $0.02 and $0.80
c. $0.025 and $0.1
d. $0.10 and $0.02

1
6. An industry is comprised of 20 firms, each with an equal market share. What is the four-firm
concentration ratio of this industry?
a. 0.2
b. 0.4
c. 0.6
d. 0.8
7. An industry has a Lerner index of zero. This number:
a. reveals that social welfare would be improved by regulating the firms.
b. is consistent with the industry being monopolistically competitive.
c. is consistent with the industry being perfectly competitive.
d. reveals that social welfare would be improved by regulating the firms and is consistent
with the industry being monopolistically competitive.
Note: in a perfectly competitive market (where the Lerner index is zero), firms already operate
at the efficient level of output where price equals marginal cost. Therefore, there would be no
need for regulation to improve social welfare further.
8. Monopolistic competition is characterized by:
a. heterogeneous products.
b. employing labor from a perfectly competitive labor market.
c. no free entry.
d. large markets.
9. In perfect competition, which is NOT true?
a. Every firm has a small but perceivable market power.
b. There are a large number of firms.
c. Firms are price-takers.
d. Firms produce homogenous goods.
e. Both concentration ratios and Rothschild indexes tend to be close to zero.
Suppose that there are two industries, A and B. There are five firms in industry A with sales at
$5 million, $2 million, $1 million, $1 million, and $1 million, respectively. There are four firms
in industry B with equal sales of $2.5 million for each firm. Answer questions (10 – 11)
10. The four-firm concentration ratio and the HHI for industry A is:
a. CR4 = 0.9; HHI = 3200
b. CR4 = 1.0; HHI = 2500
c. CR4 = 0.8; HHI = 3200
d. CR4 = 0.9; HHI = 1800
11. The four-firm concentration ratio and the HHI for industry B is:
a. CR4 = 0.9; HHI = 3200
b. CR4 = 1.0; HHI = 2500

2
c. CR4 = 0.8; HHI = 3200
d. CR4 = 0.9; HHI = 1800
12. A student figured out that the HHI for an industry was 15,000. What is the proper
conclusion?
a. The market is monopolistic.
b. The market is close to perfectly competitive or monopolistically competitive.
c. The student made some computational errors.
d. There is free entry in the market.

Suppose each of the 50 states in the U.S had only one gasoline station, and all stations were the
same size. Answer questions (13 – 14)
13. The four-firm concentration ratio, based on national data, would be:
a. 0.08.
b. 0.16.
c. 0.32.
d. 1.0.
14. The four-firm concentration ratio for the state of New York, based on the state data, is:
a. 1.0. as there is only one firm operating in the state of New York
b. 0.08.
c. 0.32.
d. 0.16.

15. A local telephone company charges $.10/min. based on a $.08/min. marginal cost of
operation. What is the Lerner index?
a. 0.2
b. 0.25
c. 0.40
d. 0.50
16. Which of the following integration types has the potential problem of increasing the firm's
market power?
a. Vertical integration
b. Horizontal integration
c. Cointegration
d. Conglomerate integration

3
17. The industry elasticity of demand for gadgets is −2, while the elasticity of demand for an
individual gadget manufacturer's product is −10. Based on the Rothschild approach to
measuring market power, we conclude that:
a. the Herfindahl index for this industry is 5.
b. the Herfindahl index for this industry is 0.2.
c. there is no monopoly power in this industry.
d. there is significant monopoly power in this industry
18. The tobacco industry has a Lerner index of 0.76. Based on this information, compute the
markup factor.
a. 4.17 times price
b. 4.17 times marginal cost
c. 0.24 times price
d. There is not sufficient information to determine the markup factor.
19. The Lerner index in the paper industry is 0.58. Based on this information, a firm charging
$3.25 per ream of paper should have a marginal cost of:
a. $0.
b. $1.365.
c. $1.885.
d. $3.25.
20. Holding all else constant, higher prices will:
a. increase the Lerner index.
b. decrease the Lerner index.
c. have no impact on the Lerner index.
d. increase or decrease the Lerner index depending on the relative magnitude of the price
increase.
21. Having worked for many of the firms in the petroleum industry, you know that the price
elasticity of demand for a representative firm is about −1.25. An industry publication recently
reported that the Rothschild index for the petroleum industry is estimated to be 0.88. Based on
this information, you know that the price elasticity of demand for the firm you currently work
for in the petroleum industry is:
a. −1.42.
b. −1.10.
c. 0.704.
d. 1.10.

4
22. Suppose you read in an industry publication that the Rothschild index for the petroleum
industry is 0.88. Based on past experience, you know that the price elasticity of demand for
the petroleum products sold by your firm is −1.5. Based on this information, you know that
the elasticity of demand for a representative firm in the petroleum industry is:
a. 1.32.
b. 1.70.
c. −0.587.
d. −1.32.
23. Suppose that the demand in a particular industry is given by Qd = 100 – 2P. When the
market price in the industry is $10 per unit, total demand in the industry is __ __.
Furthermore, assume that each of the four largest firms in the industry sell 15 units. Based on
this information, the four-firm concentration ratio is ____ .
a. 80 units; 1.00
b. 45 units; 0.75
c. 80 units; 0.75
d. 45 units; 0.25
24. Suppose the market for good X has a four-firm concentration ratio of 0.80. Having
worked for the four largest firms in the industry, you know the sales for these four firms are
given by $100,000, $125,000, $150,000, and $175,000. Based on this information, we know
that sales for the remaining firms in the industry are:
a. $687,500.
b. $550,000.
c. $250,500.
d. $137,500.
Answer
100000 + 125000 + 150000 + 175000
= 0.8
𝑇𝑜𝑡𝑎𝑙 𝑀𝑎𝑟𝑘𝑒𝑡 𝑆𝑎𝑙𝑒𝑠
550000
= 0.8
𝑇𝑜𝑡𝑎𝑙 𝑀𝑎𝑟𝑘𝑒𝑡 𝑆𝑎𝑙𝑒𝑠

𝑇𝑜𝑡𝑎𝑙 𝑀𝑎𝑟𝑘𝑒𝑡 𝑆𝑎𝑙𝑒𝑠 = 687500


𝑆𝑎𝑙𝑒𝑠 𝑓𝑜𝑟 𝑡ℎ𝑒 𝑟𝑒𝑚𝑎𝑖𝑛𝑖𝑛𝑔 𝑓𝑖𝑟𝑚𝑠 = 687500 − 550000 = 1375000

5
25. Suppose that the demand in a particular industry is given by Qd = 500 − 2P. When the
market price in the industry is $50 per unit, total demand in the industry is _________.
Furthermore, assume that the entire market consists of four firms that share the market
equally. The HHI under these conditions is then _________.
a. 225 units; 1,600
b. 400 units; 2,500
c. 225 units; 3,333.33
d. 400 units; 10,000
26. Four firms control the market for a particular good, resulting in an HHI of 2,900. Total
industry sales are $500, and it is known that two firms each have sales of $175. If each of the
remaining two firms have the same sales, then we can conclude that the remaining two firms
each have a market share of:
a. $125.
b. $75.
c. 0.15.
d. 0.50.
Answer
Market share for each of the given two firms = 175/500 = 0.35
Market share for both of them = 0.35 * 2 = 0.7
Market share for the remaining two firms = 1 – 0.7 = 0.3
Market share for each of them = 0.3/2 = 0.15
27. Which of the following is NOT considered a measure of firm conduct?
a. Lerner index of pricing behavior
b. Research and development measures
c. Advertising measures
d. Dansby-Willig index

28. An industry consists of five firms with equal annual sales. What is the industry's HHI?
a. 2,000
b. 2,500
c. 10,000
d. There is not sufficient information to compute the industry HHI.

6
29. There are five firms in an industry. You know sales of the four largest firms are
$800,000, $700,000, $440,000, and $230,000. If the C4 ratio is 80 percent, then the HHI is:
a. 1,810.
b. 2,271.
c. 4,338.
d. 5,191.
Answer
Total market sales = 2712500
Sales of the remaining firm = 542500
HHI = 2270.84
30. What is implied when the total cost of producing Q1 and Q2 together is less than the total
cost of producing Q1 and Q2 separately?
a. Economies of scale
b. Diminishing average fixed costs
c. Cost complementarity
d. Economies of scope

31. Which of the following cost functions exhibits economies of scope when three (3) units of
good one and two (2) units of good two are produced?

31. Answer (A)


32. Economies of scope exist when:

32. Answer (C)


33. Two firms producing identical products may merge due to the existence of:
a. economies of scope.
b. economies of scale.
c. cost complementarities.
d. All the preceding statements are correct.

7
34. Which of the following cost functions exhibits cost complementarity?

34. Answer (A)


35. Cost complementarity exists in a multiproduct cost function when:
a. the average cost of producing one output is reduced when the output of another product is
increased.
b. the average cost of producing one output is increased when the output of another product
is increased.
c. the marginal cost of producing one output is increased when the output of another
product is decreased.
d. the marginal cost of producing one output is reduced when the output of another product
is increased.
36. Which of the following cost functions exhibits cost complementarity?

36. Answer (D)


A nightclub manager realizes that the demand for drinks is more elastic among students, and is
trying to determine the optimal pricing schedule. He estimates that students (under age 21) have
demand curve q = 18-5p, while non-students have demand curve q = 10-2p. The two groups visit
the club in equal proportions. Producing each drink costs the nightclub $2. Answer question (37)
37. What is the uniform profit maximizing quantity and price for the monopolist?
a. q = 3; p = 3.5
b. q = 3; p = 7
c. q = 7; p = 3
d. q = 4; p = 2.8
Now suppose the nightclub can charge based on age but is limited to linear pricing. Answer
questions (38 – 39) Third degree price discrimination
38. What is the profit maximizing quantity and price for students?
a. q = 3; p = 3.5

8
b. q = 3; p = 7
c. q = 7; p = 3
d. q = 4; p = 2.8
39. What is the profit maximizing quantity and price for non-students?
a. q = 3; p = 3.5
b. q = 3; p = 7
c. q = 7; p = 3
d. q = 4; p = 2.8
The manager of a local movie theater believes that demand for a film depends on when the
movie is shown. Early moviegoers who go to films before 5pm are more sensitive to price than
are evening moviegoers. With some market research the manager discovers that the demand
curves for their daytime (d) and evening (e) moviegoers are
Qd = 100 − 10Pd and Qe = 140 − 10Pe
respectively. The marginal cost of showing a movie is constant and equal to 3 euros per customer
no matter when the movie is shown. if the manager charges the same price for daytime and
evening attendance Answer questions (40 – 44)
40. What is the profit maximizing quantity and price?
a. q = 65; p = 25
b. q = 25; p = 12
c. q = 90; p = 7.5
d. q = 7.5; p = 90
41. What is the attendance for daytime showing?
a. 65.
b. 25.
c. 90.
d. 7.5.
42. What is the attendance for evening showing?
a. 65.
b. 25.
c. 90.
d. 7.5.
43. The profits of the movie theater by using a uniform price:
a. 65.
b. 425.
c. 90.
d. 405.

9
44. What is the total welfare under uniform price?
a. 211.25
b. 647.5
c. 151.25
d. 637.5
Answer

Now suppose that the manager adopts a third-degree price discrimination scheme, setting a
different day and evening price. Answer questions (45 – 48)
45. What are the profit maximizing price and attendance for evening showing?
a. P = 6.5 Q = 55
b. P = 8.5 Q = 35
c. P = 8.5 Q = 55
d. P = 6.5 Q = 35
46. What are the profit maximizing price and attendance for daytime showing?
a. P = 6.5 Q = 55
b. P = 8.5 Q = 35
c. P = 8.5 Q = 55
d. P = 6.5 Q = 35
47. Under third-degree price discrimination the profits are:
a. 65.
b. 425.
c. 90.
d. 405.

10
48. What is the total welfare under third degree price discrimination?
a. 211.25
b. 647.5
c. 151.25
d. 637.5

Answer

49. Consider a monopolist with demand: Q = 120 – 2P and marginal cost MC = 40. Determine
the optimal quantity, profit and consumer surplus under perfect price discrimination: First degree
a. Q = 60 Profit = 120 C.S = 0
b. Q = 40 Profit = 120 C.S = 0
c. Q = 40 Profit = 400 C.S = 0
d. Q = 60 Profit = 400 C.S = 0

A monopoly is facing two groups of consumers. Consumers of type 1 have relatively


low demand given by PL = 5 − qL and consumers of type 2 have relatively high
demand PH = 10 − qH . The firm’s marginal cost is MC = 0.

Answer questions (50 – 53)


50. Suppose the types are observable and the firm offers two packages with quantity qi at
fixed monthly fee Fi what quantities and fees maximize the firm’s profits?
a. For consumers of type 1 the package is qL = 5, FL = 0; for consumers of type 2
the package is qH = 10, FH = 0.
b. For consumers of type 1 the package is qL = 5, FL = 12.5; for consumers of type
2 the package is qH = 10, FH = 50.
c. For consumers of type 1 the package is qL = 5, FL = 50; for consumers of type 2
the package is qH = 10, FH = 12.5.
d. Insufficient information.

11
51. Now suppose that the firm is unable to distinguish the types, would the pricing scheme from
question (50) work if the firm kept offering the same two packages?
a. It would work for consumers of type (1) but it wouldn’t work for consumers of type (2).
b. It would work for consumers of type (2) but it wouldn’t work for consumers of type (1).
c. It would work for both types of consumers.
d. It would work for neither type of consumers.
Answer: Consumers of type 1 keep buying their package (for them the other package
does not look attractive at all), but consumers of type 2 prefer the package designed for
type 1: Because if they consume 5 units they would pay $37.5 (area under their D-curve
for q=5), but only pay 12.5 dollars for the package designed for type 1, which leaves
them with a positive consumer surplus ( 37.5 – 12.5 = $25), whereas their own package
results in CS = 0
52. If the firm is unable to distinguish between the types, but wants to offer two different
packages with quantities you found in question (50), what monthly fees should it charge to the
different types of consumers?
a. For consumers of type 1 FL = $12.5; for consumers of type 2 FH = $50.
b. For consumers of type 1 FL = $12.5; for consumers of type 2 FH = $37.5.
c. For consumers of type 1 FL = $12.5; for consumers of type 2 FH = $25.
d. For consumers of type 1 FL = $12.5; for consumers of type 2 FH = $12.5.
Answer: If the firm wants to sell qL = 5 and qH = 10 respectively, it should lower the FH
by $25 so that the customers of this type get at least as high CS as when they buy the
other package.
53. Does the monopolist maximize their profits from quantities you found in question (50) in
case of heterogenous consumers? Try to explain your answer
a. Yes.
b. No.
Answer: The monopolist can increase their profits, in the case of heterogeneous consumers, by
charging a price above marginal cost for low-demand consumers and producing below the
competitive quantity in the low-demand market.

12
True or False:
1. Third-degree price discrimination always reduces economic efficiency, relative to what
would be achieved by a single, uniform monopoly price.
False. Third-degree price discrimination can improve the social welfare when the total output
produced under the third-degree price discrimination is greater than under a uniform price.
This situation occurs when under uniform pricing only one market is active. If the
monopolistic firm adopts third-degree price discrimination, the inactive market is served and
the total output increases with respect to the case in which it sets a uniform price.
2. A monopoly never provides an efficient level of output on its own.
False. The efficient level of output can be obtained when the monopolistic firm adopts the
first-degree price discrimination. In this case, the price is set equal to the marginal cost as
under perfect competition. The consumers will also pay a fee that is exactly equal to their
consumer surplus. (Notice that this is just a transfer from the consumers to the firm)
Therefore, the monopolist is able to extract the entire consumer surplus, and, at the same
time, it produces the efficient amount of output achieved under perfect competition, without
losses in efficiency.
3. the retail gasoline market consists of six firms. Firm 1 has 35 percent of the market, Firm
2 has 25 percent, and the remaining firms have 10 percent each. Then, Four-firm
concentration ratio = 0.65. (False, C4 = 0.8)
4. The widget industry is comprised of six firms of varying sizes. Firm 1 has 35 percent of
the market. Firm 2 has 25 percent, and the remaining firms have 10 percent each. Then,
the Herfindahl-Hirschman index for the widget industry = 1875. (False, 2250)

13

You might also like